Mathcenter Forum  

Go Back   Mathcenter Forum > คณิตศาสตร์ทั่วไป > ปัญหาคณิตศาสตร์ทั่วไป
สมัครสมาชิก คู่มือการใช้ รายชื่อสมาชิก ปฏิทิน ข้อความวันนี้

ตั้งหัวข้อใหม่ Reply
 
เครื่องมือของหัวข้อ ค้นหาในหัวข้อนี้
  #46  
Old 05 มกราคม 2005, 12:19
warut warut ไม่อยู่ในระบบ
กระบี่ไร้สภาพ
 
วันที่สมัครสมาชิก: 24 พฤศจิกายน 2001
ข้อความ: 1,627
warut is on a distinguished road
Smile

สำหรับ upper bound ที่ดีที่สุดของโจทย์ข้อ 14(ii) เนี่ยสามารถหาได้โดยการ
หาค่าสูงสุดของ abc ภายใต้เงื่อนไขว่า a + b + c + abc = 1 และ a, b, c > 0
ซึ่งผมหาโดยใช้ calculus ได้ผลคือ ค่าสูงสุดจะเกิดขึ้นเมื่อ a = b = c (ซึ่งก็ไม่น่า
แปลกใจเพราะสมการทุกอันมีความสมมาตรหมดเลย) ดังนั้นเราจะได้ว่า
3a + a3 = 1 นั่นคือ
\[a=b=c=\sqrt[3]{\frac{1+\sqrt5}{2}}+\sqrt[3]{\frac{1-\sqrt5}{2}}
=0.3221853546\dots\]
ดังนั้นค่าสูงสุดคือ abc = a3 = 1 - 3a
สรุปได้ว่า
\[abc\le1-3\left(\sqrt[3]{\frac{1+\sqrt5}{2}}+\sqrt[3]{\frac{1-\sqrt5}{2}}\right)=0.0334439361\dots\approx\frac{1}{29.9}\]
ดีกว่า upper bound ที่คุณ aaaaให้ไว้ (1/27) อีกเล็กน้อยครับ

05 มกราคม 2005 13:24 : ข้อความนี้ถูกแก้ไขแล้ว 2 ครั้ง, ครั้งล่าสุดโดยคุณ warut
ตอบพร้อมอ้างอิงข้อความนี้
  #47  
Old 05 มกราคม 2005, 22:44
warut warut ไม่อยู่ในระบบ
กระบี่ไร้สภาพ
 
วันที่สมัครสมาชิก: 24 พฤศจิกายน 2001
ข้อความ: 1,627
warut is on a distinguished road
Smile

รู้สึกว่าโจทย์ปีใหม่ของคุณ nooonuii จะยังเหลือข้อ 7 กับข้อ 10 เท่านั้นนะครับ
ที่ยังขายไม่ออก ที่ขายดีช่วงนี้เห็นจะมีแต่อสมการ (เขี่ยผมตกขอบจักรวาลไปเลย )

สำหรับข้อ 10 เนี่ยผมว่าเป็นโจทย์ที่น้องๆน่าจะทำได้นะ ตอนนี้เท่าที่ผมคิดได้คือ
ทำอะไรคล้ายๆแบบเนี๊ยะ...

ถ้าวันที่ 1 มี.ค. เป็นวันจันทร์ จะได้วันที่ 13 ส.ค. เป็นวันศุกร์
ถ้าวันที่ 1 มี.ค. เป็นวันอังคาร จะได้วันที่ 13 พ.ค. เป็นวันศุกร์
.
.
.
ตอบพร้อมอ้างอิงข้อความนี้
  #48  
Old 05 มกราคม 2005, 23:11
nooonuii nooonuii ไม่อยู่ในระบบ
ผู้พิทักษ์กฎทั่วไป
 
วันที่สมัครสมาชิก: 25 พฤษภาคม 2001
ข้อความ: 6,408
nooonuii is on a distinguished road
Post

งั้นผมขอเปลี่ยนแนวเป็นโจทย์พีชคณิตดูบ้างนะครับ

15. ให้ \( x_{1}, x_{2},...,x_{2548} \) เป็นรากของสมการพหุนาม
\( x^{2548} + x^{2547} + ... + x + 1 = 0 \) จงหาค่าของ

\[ \frac{1}{1-x_{1} } +\frac{1}{1-x_{2} }+...+\frac{1}{1-x_{2548} } \]
__________________
site:mathcenter.net คำค้น
ตอบพร้อมอ้างอิงข้อความนี้
  #49  
Old 05 มกราคม 2005, 23:47
aaaa's Avatar
aaaa aaaa ไม่อยู่ในระบบ
ลมปราณบริสุทธิ์
 
วันที่สมัครสมาชิก: 01 มกราคม 2005
ข้อความ: 109
aaaa is on a distinguished road
Post

ให้ \( y=1/(1-x) \) ดังนั้น \( x=1-1/y \) และได้พหุนามในเทอมของ \( y \)
\[
p(y)=\left(1-\frac{1}{y}\right)^{2548}+\left(1-\frac{1}{y}\right)^{2547}+\cdots+1,\qquad
y^{2548}p(y)=y^{2548}\left(1-\frac{1}{y}\right)^{2548}+y^{2548}\left(1-\frac{1}{y}\right)^{2547}+\cdots+y^{2548}
\]
สปสของเทอม \( y^{2548} \) คือ 2549 และสปสของเทอม \( y^{2547} \) เท่ากับ
\[
-\left({2548\choose1}+{2547\choose1}+\cdots+{1\choose1}\right)=-\frac{2548\times2549}{2}
\]
ดังนั้น
\[
y_1+y_2+\cdots+y_{2548}=2548/2=1274
\]
ตอบพร้อมอ้างอิงข้อความนี้
  #50  
Old 06 มกราคม 2005, 00:45
nooonuii nooonuii ไม่อยู่ในระบบ
ผู้พิทักษ์กฎทั่วไป
 
วันที่สมัครสมาชิก: 25 พฤษภาคม 2001
ข้อความ: 6,408
nooonuii is on a distinguished road
Post

เยี่ยมครับคุณ aaaa
นี่คือวิธีคิดของผม ซึ่งเป็นวิธีคิดอีกแบบนึงครับ

ก่อนอื่นสังเกตว่า \( x_{i} \) คือรากที่ 2549 ของ 1 แต่ไม่เท่ากับ 1 ดังนั้น \( x_{i} \) ทั้งหมดเป็นจำนวนเชิงซ้อนที่ไม่ใช่จำนวนจริง
ดังนั้น เราสามารถจับคู่ \( x_{i} \) ซึ่งเป็น conjugate กันได้ทั้งหมด 1274 คู่
โดยการสังเกตว่า \[ \frac{1}{1-z} + \frac{1}{1-\overline{z}} = 1 \] เมื่อ |z|=1 และ z1 เราจะได้ว่า
\[ \frac{1}{1-x_{1}} + \frac{1}{1-x_{2}} + ... + \frac{1}{1-x_{2548}} = 1274 \]
__________________
site:mathcenter.net คำค้น

06 มกราคม 2005 00:51 : ข้อความนี้ถูกแก้ไขแล้ว 2 ครั้ง, ครั้งล่าสุดโดยคุณ nooonuii
ตอบพร้อมอ้างอิงข้อความนี้
  #51  
Old 06 มกราคม 2005, 01:14
nooonuii nooonuii ไม่อยู่ในระบบ
ผู้พิทักษ์กฎทั่วไป
 
วันที่สมัครสมาชิก: 25 พฤษภาคม 2001
ข้อความ: 6,408
nooonuii is on a distinguished road
Post

16. ให้ \( P(x) = x^{2548} + a_{2547}x^{2547} + ...+a_{1}x + a_{0} \)
โดยที่
\[ P(0) = 1, P(1) = \frac{1}{2} , ... , P(2547) = \frac{1}{2548} \]
จงหาค่าของ P(2548)
__________________
site:mathcenter.net คำค้น

06 มกราคม 2005 01:20 : ข้อความนี้ถูกแก้ไขแล้ว 1 ครั้ง, ครั้งล่าสุดโดยคุณ nooonuii
ตอบพร้อมอ้างอิงข้อความนี้
  #52  
Old 06 มกราคม 2005, 01:50
aaaa's Avatar
aaaa aaaa ไม่อยู่ในระบบ
ลมปราณบริสุทธิ์
 
วันที่สมัครสมาชิก: 01 มกราคม 2005
ข้อความ: 109
aaaa is on a distinguished road
Post

เฉลยข้อ 16
ให้ \( Q(x)=(x+1)P(x)-1 \) ดังนั้น \( 0,1,2,\ldots,2547 \) เป็นรากของ \( Q(x) \) แต่เนื่องจากมันมี degree 2549 และเป็น monic polynomial ดังนั้น
\[
Q(x)=x(x-1)(x-2)\cdots(x-2547)(x-k)
\]
แทนค่า \( Q(-1)=(-1+1)P(-1)-1=-2548!(k+1) \) ดังนั้น \( k=1/2548!-1 \) แทนค่า \( x=2548 \) ใน \( Q(x) \) ได้คำตอบตามต้องการคือ
\( P(2548)=2548! \)

PS ขอบคุณ คุณ warut ครับ ข้อนี้น่าจะมาจากข้อสอบ USAMO

07 มกราคม 2005 08:25 : ข้อความนี้ถูกแก้ไขแล้ว 2 ครั้ง, ครั้งล่าสุดโดยคุณ aaaa
ตอบพร้อมอ้างอิงข้อความนี้
  #53  
Old 06 มกราคม 2005, 04:39
nooonuii nooonuii ไม่อยู่ในระบบ
ผู้พิทักษ์กฎทั่วไป
 
วันที่สมัครสมาชิก: 25 พฤษภาคม 2001
ข้อความ: 6,408
nooonuii is on a distinguished road
Post

ลองโจทย์เรขาคณิตบ้างครับ

17. ให้ \( \large{P_{1},...,P_{2548}} \) เป็นจุดยอดของรูป 2548 เหลี่ยมด้านเท่าที่แนบในวงกลมหนึ่งหน่วย ให้ \( \large{ r_{i}} \) แทนความยาวของส่วนของเส้นตรงที่ลากเชื่อมจุด P1 กับ Pi , i = 1,2,...,2548 จงหาค่าของ
\[ \large{r_{2}r_{3}...r_{2548}} \]
__________________
site:mathcenter.net คำค้น

06 มกราคม 2005 04:43 : ข้อความนี้ถูกแก้ไขแล้ว 1 ครั้ง, ครั้งล่าสุดโดยคุณ nooonuii
ตอบพร้อมอ้างอิงข้อความนี้
  #54  
Old 06 มกราคม 2005, 04:47
aaaa's Avatar
aaaa aaaa ไม่อยู่ในระบบ
ลมปราณบริสุทธิ์
 
วันที่สมัครสมาชิก: 01 มกราคม 2005
ข้อความ: 109
aaaa is on a distinguished road
Post

(BMO)
จงหาคู่อันดับ \( (r,s) \) ที่เป็นจำนวนเต็มทั้งหมดซึ่ง \( 0<s<200 \) และ
\[ \frac{59}{80}<\frac{r}{s}<\frac{45}{61} \]

07 มกราคม 2005 08:26 : ข้อความนี้ถูกแก้ไขแล้ว 2 ครั้ง, ครั้งล่าสุดโดยคุณ aaaa
ตอบพร้อมอ้างอิงข้อความนี้
  #55  
Old 06 มกราคม 2005, 10:24
nooonuii nooonuii ไม่อยู่ในระบบ
ผู้พิทักษ์กฎทั่วไป
 
วันที่สมัครสมาชิก: 25 พฤษภาคม 2001
ข้อความ: 6,408
nooonuii is on a distinguished road
Post

18. จงหาพหุนาม P(x) ทั้งหมดซึ่งสอดคล้องคุณสมบัติ
\[ P(\frac{x+y}{2}) = \frac{P(x)+P(y)}{2} \]
ทุกจำนวนจริง x,y
__________________
site:mathcenter.net คำค้น
ตอบพร้อมอ้างอิงข้อความนี้
  #56  
Old 07 มกราคม 2005, 00:50
warut warut ไม่อยู่ในระบบ
กระบี่ไร้สภาพ
 
วันที่สมัครสมาชิก: 24 พฤศจิกายน 2001
ข้อความ: 1,627
warut is on a distinguished road
Post

ผมว่าเฉลยข้อ 16 ของคุณ aaaa จบลงแบบค้างๆอยู่นา ไม่ทราบว่าตั้งใจจะให้
เป็นเช่นนั้นรึเปล่า ถ้าใช่ผมก็ขอต่อให้จบว่า P(2548) = 2548! ครับ

อ้อ...รู้สึกว่าคุณ aaaa จะพิมพ์ค่า k ผิดไป ที่ถูกน่าจะเป็น 1/2548! - 1 นะครับ
ตอบพร้อมอ้างอิงข้อความนี้
  #57  
Old 07 มกราคม 2005, 01:22
warut warut ไม่อยู่ในระบบ
กระบี่ไร้สภาพ
 
วันที่สมัครสมาชิก: 24 พฤศจิกายน 2001
ข้อความ: 1,627
warut is on a distinguished road
Smile

อ้างอิง:
ข้อความเดิมของคุณ nooonuii:
7. จงพิสูจน์โดยไม่ใช้ Fermat's Last Theorem ว่า ไม่มีจำนวนเฉพาะ p,q,r ซึ่งสอดคล้องสมการ pn + qn = rn ทุกค่า n2
เฉลย: โดยพิจารณาจาก parity (เลขคู่-เลขคี่) จะเห็นว่าใน p, q, r นั้นจะ
ต้องมีตัวหนึ่งเท่ากับ 2 และที่เหลืออีก 2 ตัวจะต้องเป็นจำนวนเฉพาะคี่
ดังนั้นสิ่งที่จะต้องทำก็คือการแก้สมการ Diophantine
\[q^n=p^n+2^n\]
จากสมการจะเห็นว่า q > p
แต่ q < p + 2 เพราะ pn + 2n < (p + 2)n
ดังนั้น q = p + 1 แต่ถ้า p เป็นจำนวนเฉพาะคี่แล้ว p + 1 จะไม่ใช่
จำนวนเฉพาะ สมการนี้จึงบ่มีคำตอบเด้อ

07 มกราคม 2005 01:39 : ข้อความนี้ถูกแก้ไขแล้ว 1 ครั้ง, ครั้งล่าสุดโดยคุณ warut
ตอบพร้อมอ้างอิงข้อความนี้
  #58  
Old 07 มกราคม 2005, 01:53
nooonuii nooonuii ไม่อยู่ในระบบ
ผู้พิทักษ์กฎทั่วไป
 
วันที่สมัครสมาชิก: 25 พฤษภาคม 2001
ข้อความ: 6,408
nooonuii is on a distinguished road
Post

สุดยอดเลยครับ คุณ warut แหมนึกว่าจะไม่มีใครตอบโจทย์ข้อนี้ซะแล้ว อีกอย่างวิธีคิดของคุณ warut นี่สั้นกระชับดีมากเลยครับ ของผมค่อนข้างยาวและมั่วกว่านี้ครับ

เสริมอีกนิดครับว่าเราใช้เงื่อนไข n2 ตรงที่เราอ้างว่า pn + 2n < (p + 2)n ครับ

ขอเล่าที่มาของโจทย์ข้อนี้นิดนึงครับ เทอมที่แล้วผมหันมาสนใจทฤษฎีบทสุดท้ายของ Fermat ด้วยความบ้าระห่ำ ฮิฮิ เลยเริ่มคิดโจทย์ข้อนี้ขึ้นมาก่อน จากนั้นก็ตั้งใจว่าจะขยายไปสู่กรณีทั่วไปที่ Wiles พิสูจน์ไว้แล้วโดยจะเอาพีชคณิตมาเล่น แต่ตอนนี้รู้สึกว่าจะติดขัดกลายเป็นว่าสิ่งที่ผมจะพิสูจน์มันกลายเป็นงูกินหางไป ผมก็เลยพักไว้ก่อนแล้วหันมาอ่านหนังสือเตรียมสอบ qualify แทน คิดว่าถ้าหัวปลอดโปร่งแล้วอาจจะมานั่งคิดเรื่องนี้ต่อครับ

P.S. สมการที่คุณ warut ยกมา ถ้าให้ n=1 จะลดรูปไปเป็น conjecture ที่น่าสนใจอีกอันก็คือ Twin Prime Conjecture ซึ่งน่าสนใจมากครับ น่าเสียดายที่ผมยังไม่เก่งทฤษฎีจำนวนไม่งั้นผมคงบ้าระห่ำกว่านี้
__________________
site:mathcenter.net คำค้น

07 มกราคม 2005 02:22 : ข้อความนี้ถูกแก้ไขแล้ว 2 ครั้ง, ครั้งล่าสุดโดยคุณ nooonuii
ตอบพร้อมอ้างอิงข้อความนี้
  #59  
Old 07 มกราคม 2005, 02:08
warut warut ไม่อยู่ในระบบ
กระบี่ไร้สภาพ
 
วันที่สมัครสมาชิก: 24 พฤศจิกายน 2001
ข้อความ: 1,627
warut is on a distinguished road
Smile

อ้างอิง:
ข้อความเดิมของคุณ nooonuii:
ลองโจทย์เรขาคณิตบ้างครับ

17. ให้ \( \large{P_{1},\dots,P_{2548}} \) เป็นจุดยอดของรูป 2548 เหลี่ยมด้านเท่าที่แนบในวงกลมหนึ่งหน่วย ให้ \( \large{ r_{i}} \) แทนความยาวของส่วนของเส้นตรงที่ลากเชื่อมจุด P1 กับ Pi , i = 1,2,...,2548 จงหาค่าของ
\[ \large{r_{2}r_{3}\dots r_{2548}} \]
เฉลย: เนื่องจาก
\[r_i=2\sin\frac{\left(i-1\right)\pi}{2548}\]
ดังนั้น
\[r_2r_3\dots r_{2548}=
2^{2547}\sin\frac{\pi}{2548}\sin\frac{2\pi}{2548}\dots\sin\frac{2547\pi}{2548}\]
แต่เรามีสูตรอยู่ว่า
\[\sin\frac{\pi}{n}\sin\frac{2\pi}{n}\dots\sin\frac{\left(n-1\right)\pi}{n}
=\frac{n}{2^{n-1}}\]
(ผมจำไม่ได้ว่าเคยมีคนแสดงการพิสูจน์ของสูตรนี้ไว้ในเว็บบอร์ดแห่งนี้
แล้วหรือยัง แต่ถ้าเคยมีแล้วผมก็จำไม่ได้อยู่ดีว่าเค้าพิสูจน์ไว้ยังไง )

ดังนั้นคำตอบของข้อนี้จึงเท่ากับ 2548 ครับผม
ตอบพร้อมอ้างอิงข้อความนี้
  #60  
Old 07 มกราคม 2005, 02:35
nooonuii nooonuii ไม่อยู่ในระบบ
ผู้พิทักษ์กฎทั่วไป
 
วันที่สมัครสมาชิก: 25 พฤษภาคม 2001
ข้อความ: 6,408
nooonuii is on a distinguished road
Post

ว้าว คุณ warut แสดงวิธีคิดอีกแบบให้ผมดูอีกแล้ว มาดูวิธีคิดของคนรักพีชคณิตอย่างผมบ้างดีกว่าครับ

สร้างวงกลมหนึ่งหน่วยให้มีจุดศูนย์กลางอยู่ที่จุดกำเนิดในระนาบเชิงซ้อน และ ให้จุด P1 แทนจุด (1,0) = 1 ในระนาบเชิงซ้อน

ดังนั้นเราจะได้ว่า จุด Pi ทั้งหมดแทน รากที่ 2548 ของ 1 นั่นเอง และยิ่งกว่านั้นจะได้ว่า ri = |1-Pi|
ให้ Q(z) = 1+z+...+z2547 จะได้ว่า P2,...,P2548 คือรากทั้งหมดของ Q(z) นั่นคือ Q(z) = (z - P2)...(z - P2548)
\ r2...r2548 = |1-P2|...|1-P2548| = |Q(1)| = 2548

P.S. ถ้านำวิธีคิดแบบคุณ warut ผนวกกับวิธีคิดของผม เราก็จะได้บทพิสูจน์เอกลักษณ์ตรีโกณมิติ

\[ sin \frac{\pi}{n} sin \frac{2\pi}{n} ... sin\frac{(n-1)\pi}{n} = \frac{n}{2^{n-1}}\]

ว้าว สวยงามมากๆเลยครับ
__________________
site:mathcenter.net คำค้น

07 มกราคม 2005 02:46 : ข้อความนี้ถูกแก้ไขแล้ว 4 ครั้ง, ครั้งล่าสุดโดยคุณ nooonuii
ตอบพร้อมอ้างอิงข้อความนี้
ตั้งหัวข้อใหม่ Reply


หัวข้อคล้ายคลึงกัน
หัวข้อ ผู้ตั้งหัวข้อ ห้อง คำตอบ ข้อความล่าสุด
ข้อสอบสมาคม ม.ปลายปี 2548 prachya ข้อสอบในโรงเรียน ม.ปลาย 32 30 ตุลาคม 2010 12:58
ขอถามสสวท.2548หน่อยไม่มั่นใจ Wind ข้อสอบในโรงเรียน ประถมปลาย 3 27 สิงหาคม 2007 20:37
สมาคมคณิตศาสตร์ 2548 (ม.ต้น) R-Tummykung de Lamar ข้อสอบในโรงเรียน ม.ต้น 14 06 สิงหาคม 2006 11:03
โอลิมปิกคณิตศาสตร์ 2548 รอบที่ 1 devilzoa ข้อสอบโอลิมปิก 2 20 ธันวาคม 2005 14:21
สสวท .เริ่มรับสมัครสอบ แข่งโอลิมปิกปี 2548 gon ข่าวคราวแวดวง ม.ปลาย 3 29 พฤษภาคม 2004 20:40


กฎการส่งข้อความ
คุณ ไม่สามารถ ตั้งหัวข้อใหม่ได้
คุณ ไม่สามารถ ตอบหัวข้อได้
คุณ ไม่สามารถ แนบไฟล์และเอกสารได้
คุณ ไม่สามารถ แก้ไขข้อความของคุณเองได้

vB code is On
Smilies are On
[IMG] code is On
HTML code is Off
ทางลัดสู่ห้อง


เวลาที่แสดงทั้งหมด เป็นเวลาที่ประเทศไทย (GMT +7) ขณะนี้เป็นเวลา 15:37


Powered by vBulletin® Copyright ©2000 - 2024, Jelsoft Enterprises Ltd.
Modified by Jetsada Karnpracha